Đến nội dung

Hình ảnh

Bài kiểm tra Trường Đông Toán học 2016 - Viện Toán học Hà Nội


  • Please log in to reply
Chủ đề này có 16 trả lời

#1
huykinhcan99

huykinhcan99

    Sĩ quan

  • Điều hành viên OLYMPIC
  • 336 Bài viết

Câu 1 (5 điểm). Ký hiệu $x_n$ là nghiệm dương duy nhất của phương trình

\[x^n+x^{n-1}+\ldots+x=n+2\]

Chứng minh rằng dãy $\left(x_n\right)$ hội tụ tới một số thực dương. Tìm giới hạn đó.

 

Câu 2 (5 điểm). Tìm tất cả các hàm số $f:\mathbb{Q} \to \mathbb{Q}$ thoả mãn: với mọi $x\in \mathbb{Q}$, ta có đồng thời các đẳng thức sau:

  1. $f\left(x+1\right)=f\left(x\right)+1$;
  2. $f\left(x^3\right)=\left(f\left(x\right)\right)^3$.

Câu 3 (5 điểm). Cho đường tròn $\left(O\right)$ và hai điểm $B$, $C$ cố định trên đó. Điểm $A$ thay đổi trên $\left(O\right)$ sao cho tam giác $ABC$ nhọn. Phân giác trong góc $\angle BAC$ cắt $BC$ và $\left(O\right)$ lần lượt tại $D$ và $E$. Trên đường thẳng $BC$ lấy điểm $F$ sao cho $FD=FE$.

  1. Gọi $H$ là hình chiếu của $A$ trên $EF$. Chứng minh $H$ thuộc một đường tròn cố định.
  2. Ký hiệu $(I)$ là đường tròn tiếp xúc với các tia $AB$, $AC$ và đường thẳng $EF$ lần lượt tại $M$, $N$ và $P$ (tâm $I$ cùng phía với $A$ đối với đường thẳng $EF$). Lấy điểm $Q$ trên $MN$ sao cho $PQ\perp EF$. Chứng minh rằng $AQ$ đi qua một điểm cố định.

Câu 4 (5 điểm). Tại Trường Đông Toán học 2016, các thầy cô đưa ra tổng cộng $100$ bài toán cho học sinh giải, trong đó có một số bài được các thầy cô cho là khó và gọi là các bài "khoai". Bất ngờ là mỗi bài (dù "khoai" hay không "khoai") đều có đúng $20$ em giải được. Để tổng kết Trường Đông, ban tổ chức sẽ chọn ra một số em để trao giấy khen. Với một cách chọn như vậy, một bài toán sẽ được gọi là "củ chuối" nếu nó là một bài "khoai" nhưng cả $20$ em giải được nó đều không được giấy khen, hoặc nếu nó không phải "khoai " nhưng cả $20$ em giải được nó đều được giấy khen. Chứng minh rằng ban tổ chức có thể chọn ra một số học sinh để trao giấy khen sao cho trong số $100$ bài toán đưa ra có không quá $4$ bài "củ chuối".

 

Câu 5 (6 điểm). Tồn tại hay không $2016$ số nguyên tố phân biệt $p_1$, $p_2$, $\ldots$, $p_{2016}$ và số nguyên dương $n$ mà

\[\sum^{2016}_{i=1} \dfrac{1}{p^2_i+1} = \dfrac{1}{n}?\]

 

Câu 6 (7 điểm). Cho tam giác $ABC$ nhọn. Đường tròn $(I)$ nội tiếp tam giác tiếp xúc với các cạnh $BC$, $CA$, $AB$ lần lượt tại $D$, $E$, $F$. Gọi $H$ là trực tâm tam giác $DEF$; hạ $HK\perp BC$.

  1. Gọi $J$ là trung điểm $EF$. Chứng minh rằng $AD$, $KJ$ cắt nhau tại một điểm trên đường tròn $(T)$ ngoại tiếp tam giác $DHK$.
  2. Cho $AD$ cắt $(I)$ tại $P$. Trên các đường thẳng $DE$, $DF$ lần lượt lấy các điểm $M$, $N$ sao cho $\angle MPE=\angle NPF=90^\circ$. Chứng minh rằng $DJ$ là trục đẳng phương của đường tròn $(T)$ và đường tròn ngoại tiếp tam giác $MNP$.

Câu 7 (7 điểm). Tìm số thực $c$ nhỏ nhất sao cho với mọi số thực $x$, $y$, $z$ mà $x+y+z=1$, ta có bất đẳng thức

\[\left|x^3+y^3+z^3-1\right|\leqslant c\left|x^5+y^5+z^5-1\right|.\]

Với giá trị đó của $c$ thì đẳng thức xảy ra khi nào?


$$\text{Vuong Lam Huy}$$

#2
Lyness

Lyness

    Binh nhất

  • Thành viên mới
  • 29 Bài viết

Câu 1. Đặt $f_{n}(x)=x^{n}+...+x-n-2$.

Dễ thấy f là hàm đồng biến.

Ta có $f_{n}(0)=-n-2<0,f_{n}(2)>0$.

Mà hàm đồng biến trên D, nên pt có nghiệm duy nhất trên (1;2).

Ta có $f_{n+1}(x_{n+1})=0$ và $f_{n+1}(x_{n})=x_{n}^{n+1}-1>0\Rightarrow x_{n+1}<x_{n}$.

Vậy $\left \{ x_{n} \right \}$ là dãy giảm và bị dưới trên bởi 1 nên tồn tại giới hạn hữu hạn.

Đặt giới hạn đó là l. Dễ dàng tìm được l=1.


Bài viết đã được chỉnh sửa nội dung bởi Lyness: 15-12-2016 - 22:46


#3
kimchitwinkle

kimchitwinkle

    Thiếu úy

  • Điều hành viên THCS
  • 526 Bài viết

Mình xin đưa ra hướng giải quyết ở một số bài:
Câu 1: Chứng minh $x_n \in \left(1; \dfrac{3}{2}\right)$. Sau đó chứng minh $(x_n)$ giảm. Ta dự đoán giới hạn của dãy là $\lim x_n=1$. Thật vậy nếu $\lim x_n=a>1$. Đặt $a=1+\varepsilon$. Thay $x$ bởi $a$ và $n$ bởi $n-2$ vào phương trình, kết hợp rút gọn được:
$$\dfrac{(1+\varepsilon)^{n-1}-1}{\varepsilon}=n \Leftrightarrow (1+\varepsilon)^{n-1}=n\varepsilon+1$$
Ta cần chứng minh điều vô lý ở đây, theo cảm nhận ta thấy khi $n$ đủ lớn thì $VT > VP$. Ta sẽ chứng minh điều này.
Thật vậy:
$$(1+\varepsilon)^{n-1}>1+(n-1)\varepsilon+\dfrac{(n-1)(n-2)}{2}\varepsilon^2$$
Vậy ta cần chứng minh:
$$1+(n-1)\varepsilon+\dfrac{(n-1)(n-2)}{2}\varepsilon^2>n\varepsilon+1 \Leftrightarrow \dfrac{(n-1)(n-2)}{2}\varepsilon^2>\varepsilon$$
Cho $n \to + \infty$ ta có ngay bất đẳng thức trên. Vậy ta suy ra điều vô lý. Do đó $a=1$ hay $\lim x_n=1$.

Mình up các bài còn lại sau...



#4
kimchitwinkle

kimchitwinkle

    Thiếu úy

  • Điều hành viên THCS
  • 526 Bài viết

Câu 2: Đầu tiên ta tính $f(-1)$, $f(0)$ và $f(1)$ bằng cách thay lần lượt $x$ bởi $-1$, $0$, $1$ vào giả thiết (2) được $f(-1), f(0), f(1) \in \left\{-1; 0; 1\right\}$.
Nếu $f(-1)=0$, suy ra $f(0)=1$, suy ra $f(1)=2$, vô lý.
Nếu $f(-1)=1$, suy ra $f(0)=2$, vô lý.
Vậy $f(-1)=-1$, suy ra $f(0)=0$ và $f(1)=1$.
Bằng quy nạp chứng minh được $f(n)=n, \forall n \in \mathbb{Z}$.
Bây giờ mình sẽ tính $f\left(\dfrac{p}{q}\right)$ với $p, q \in \mathbb{Z}$.
Ta tính $f\left(\left(\dfrac{p}{q}+q^2\right)^3\right)$ bằng hai cách để suy ra đẳng thức sau:
$$\left(f\left(\dfrac{p}{q}\right)-\dfrac{p}{q}\right)\left(f\left(\dfrac{p}{q}\right)+\dfrac{p}{q}+q^2\right)=0 \Leftrightarrow \left[\begin{array}{lcl} f\left(\dfrac{p}{q}\right)=\dfrac{p}{q} \\ f\left(\dfrac{p}{q}\right)=-\dfrac{p}{q}-q^2\end{array}\right.$$
Giả sử tồn tại $p, q \in \mathbb{Z}$ sao cho $f\left(\dfrac{p}{q}\right)=-\dfrac{p}{q}-q^2$, từ điều kiện 1 dễ suy ra điều vô lý.
Vậy $f\left(\dfrac{p}{q}\right)=\dfrac{p}{q}$ hay $f(q)=q, \forall q \in \mathbb{Q}$.
Thử lại thỏa.


Bài viết đã được chỉnh sửa nội dung bởi kimchitwinkle: 15-12-2016 - 23:05


#5
kimchitwinkle

kimchitwinkle

    Thiếu úy

  • Điều hành viên THCS
  • 526 Bài viết

Câu 5: Ta chia hai trường hợp:
Trường hợp 1: $p_i \neq 3, \forall i=\overline{1, 2016}$. Ta có:
$$\displaystyle{\sum\limits_{i=1}^{2016}\dfrac{1}{p_i^2+1}=\dfrac{(p_2^2+1)(p_3^2+1)\cdots(p_{2016}^2+1)+(p_1^2+1)(p_3^2+1)\cdots(p_{2016}^2+1)+\cdots+(p_1^2+1)(p_3^2+1)\cdots(p_{2015}^2+1)}{(p_1^2+1)(p_2^2+1)\cdots(p_{2016}^2+1)}}$$
Xét:
$$(p_2^2+1)(p_3^2+1)\cdots(p_{2016}^2+1)+(p_1^2+1)(p_3^2+1)\cdots(p_{2016}^2+1)+\cdots+(p_1^2+1)(p_3^2+1)\cdots(p_{2015}^2+1) \equiv (-1)^{2015}\cdot2016 \equiv 0 \text{ (mod }3)$$
$$(p_1^2+1)(p_2^2+1)\cdots(p_{2016}^2+1) \equiv (-1)^{2016} \equiv 1 \text{ (mod }3)$$
Do đó không thể có dạng $\dfrac{1}{n^2}$.
Trường hợp 2: $p_1=3, p_i \neq 3, \forall i=\overline{2, 2016}$. Ta có:
$$(p_2^2+1)(p_3^2+1)\cdots(p_{2016}^2+1)+(p_1^2+1)(p_3^2+1)\cdots(p_{2016}^2+1)+\cdots+(p_1^2+1)(p_3^2+1)\cdots(p_{2015}^2+1) \equiv 1\cdot(-1)^{2014}\cdot2015+(-1)^{2015} \equiv 1 \text{ (mod }3)$$
$$(p_1^2+1)(p_2^2+1)\cdots(p_{2016}^2+1) \equiv 1\cdot(-1)^{2015} \equiv -1 \text{ (mod }3)$$
Do đó không thể có dạng $\dfrac{1}{n^2}$.
Vậy ta có điều phải chứng minh.


Bài viết đã được chỉnh sửa nội dung bởi kimchitwinkle: 15-12-2016 - 23:19


#6
kimchitwinkle

kimchitwinkle

    Thiếu úy

  • Điều hành viên THCS
  • 526 Bài viết

Mình sẽ up bài 3, 6, 7 vào ngày mai.


Bài viết đã được chỉnh sửa nội dung bởi kimchitwinkle: 15-12-2016 - 23:27


#7
pdtienArsFC

pdtienArsFC

    Trung sĩ

  • Thành viên
  • 133 Bài viết

Câu 2: Đầu tiên ta tính $f(-1)$, $f(0)$ và $f(1)$ bằng cách thay lần lượt $x$ bởi $-1$, $0$, $1$ vào giả thiết (2) được $f(-1), f(0), f(1) \in \left\{-1; 0; 1\right\}$.
Nếu $f(-1)=0$, suy ra $f(0)=1$, suy ra $f(1)=2$, vô lý.
Nếu $f(-1)=1$, suy ra $f(0)=2$, vô lý.
Vậy $f(-1)=-1$, suy ra $f(0)=0$ và $f(1)=1$.
Bằng quy nạp chứng minh được $f(n)=n, \forall n \in \mathbb{Z}$.
Bây giờ mình sẽ tính $f\left(\dfrac{p}{q}\right)$ với $p, q \in \mathbb{Z}$.
Ta tính $f\left(\left(\dfrac{p}{q}+q^2\right)^3\right)$ bằng hai cách để suy ra đẳng thức sau:
$$\left(f\left(\dfrac{p}{q}\right)-\dfrac{p}{q}\right)\left(f\left(\dfrac{p}{q}\right)+\dfrac{p}{q}+q^2\right)=0 \Leftrightarrow \left[\begin{array}{lcl} f\left(\dfrac{p}{q}\right)=\dfrac{p}{q} \\ f\left(\dfrac{p}{q}\right)=-\dfrac{p}{q}-q^2\end{array}\right.$$
Giả sử tồn tại $p, q \in \mathbb{Z}$ sao cho $f\left(\dfrac{p}{q}\right)=-\dfrac{p}{q}-q^2$, từ điều kiện 1 dễ suy ra điều vô lý.
Vậy $f\left(\dfrac{p}{q}\right)=\dfrac{p}{q}$ hay $f(q)=q, \forall q \in \mathbb{Q}$.
Thử lại thỏa.

Câu nói mang tính ngộ nhận quá bạn ơi :D Nếu có thể bạn giải thích rõ hơn nhé.

Thay vào đó, bạn có thể tính $f\left(\left(\dfrac{p}{q}-q^2\right)^3\right)$ bằng 2 cách, kết hợp với cái bạn có thì ta được đpcm nhé :P


                           80b68e1e79774daab705a98543684359.0.gif

 


#8
kimchitwinkle

kimchitwinkle

    Thiếu úy

  • Điều hành viên THCS
  • 526 Bài viết

Mình tính cho bạn xem:
Từ $f(x+1)=f(x)+1$. Mình quy nạp được $f(x+n)=f(x)+n, \forall n \in \mathbb{Z}$.
Với mọi $p, q \in \mathbb{Z}$, ta có:
$$f\left(\left(\dfrac{p}{q}+q^2\right)^3\right)=f\left(\dfrac{p^3}{q^3}+3p^2+3pq^3+q^6\right)=f^3\left(\dfrac{p}{q}\right)+3p^2+3pq^3+q^6$$
Mặt khác:
$$f\left(\left(\dfrac{p}{q}+q^2\right)^3\right)=\left[f\left(\dfrac{p}{q}+q^2\right)\right]^3=\left[f\left(\dfrac{p}{q}\right)+q^2\right]^3=f^3\left(\dfrac{p}{q}\right)+3f^2\left(\dfrac{p}{q}\right)q^2+3f\left(\dfrac{p}{q}\right)q^4+q^6$$
Trừ vế theo vế, mình được đẳng thức đã nêu. :)


Bài viết đã được chỉnh sửa nội dung bởi kimchitwinkle: 16-12-2016 - 00:07


#9
pdtienArsFC

pdtienArsFC

    Trung sĩ

  • Thành viên
  • 133 Bài viết

Mình tính cho bạn xem:
Từ $f(x+1)=f(x)+1$. Mình quy nạp được $f(x+n)=f(x)+n, \forall n \in \mathbb{Z}$.
Với mọi $p, q \in \mathbb{Z}$, ta có:
$$f\left(\left(\dfrac{p}{q}+q^2\right)^3\right)=f\left(\dfrac{p^3}{q^3}+3p^2+3pq^3+q^6\right)=f^3\left(\dfrac{p}{q}\right)+3p^2+3pq^3+q^6$$
Mặt khác:
$$f\left(\left(\dfrac{p}{q}+q^2\right)^3\right)=\left[f\left(\dfrac{p}{q}+q^2\right)\right]^3=\left[f\left(\dfrac{p}{q}\right)+q^2\right]^3=f^3\left(\dfrac{p}{q}\right)+3f^2\left(\dfrac{p}{q}\right)q^2+3f\left(\dfrac{p}{q}\right)q^4+q^6$$
Trừ vế theo vế, mình được đẳng thức đã nêu

Cái này thì tôi biết mà :(

Mình nói sau khi còn trường hợp 2 đấy, cái chỗ mình tô đỏ ấy.

Thay vào đó bạn có thể tính 1 lần nữa nhưng thay bởi dấu "-" và dùng 2 đẳng thức là được :P


                           80b68e1e79774daab705a98543684359.0.gif

 


#10
kimchitwinkle

kimchitwinkle

    Thiếu úy

  • Điều hành viên THCS
  • 526 Bài viết

Mình sẽ giải thích cách của mình: :)
Giả sử tồn tại $p, q$ sao cho $f\left(\dfrac{p}{q}\right)=-\dfrac{p}{q}-q^2$.
Suy ra $f\left(\dfrac{p+q}{q}\right)=-\dfrac{p}{q}-q^2+1$.
Vậy ta có:
$$-\dfrac{p}{q}-q^2+1=\dfrac{p+q}{q} \Leftrightarrow -\dfrac{p}{q}-q^2=\dfrac{p}{q} \Rightarrow f\left(\dfrac{p}{q}\right)=\dfrac{p}{q}$$
hoặc:
$$-\dfrac{p}{q}-q^2+1=\dfrac{-p-q}{q}-q^2 \Leftrightarrow -1=1 \text{ (vô lý)}$$
Vậy ta phải có $f\left(\dfrac{p}{q}\right)=\dfrac{p}{q}$.


Bài viết đã được chỉnh sửa nội dung bởi kimchitwinkle: 16-12-2016 - 00:23


#11
kimchitwinkle

kimchitwinkle

    Thiếu úy

  • Điều hành viên THCS
  • 526 Bài viết

Câu 3:

truongdongvientoan2016day1.jpg

a) Ta chứng minh $A$, $H$, $O$ thẳng hàng để từ đó suy ra $OH \text{ } \bot \text{ } EH$. Tức là $H$ nằm trên đường tròn đường kính $OE$.
Thật vậy, thông qua biến đổi góc:
$$\widehat{EAO}+\widehat{AEF}=\widehat{AEO}+\widehat{EDF}=90^o$$
Do đó $AO \text{ } \bot \text{ } EF \text{ } \bot \text{ } AH$. Nên $A$, $H$, $O$ thẳng hàng. Suy ra $H$ nằm trên đường tròn đường kính $OE$. Vì $O$ và $E$ cố định nên ta có điều phải chứng minh.
b) Thật ra đây là một ứng dụng từ một bổ đề khá hay đối với đường tròn nội tiếp như sau:

Bổ đề. Cho $\triangle ABC$ ngoại tiếp $(I)$. $(I)$ tiếp xúc $BC$, $CA$, $AB$ lần lượt tại $D$, $E$, $F$. $ID$ cắt $EF$ tại $G$. Khi đó $AG$ đi qua trung điểm $M$ của $BC$.
truongdongvientoan2016day1bode.jpg
Chứng minh. Kéo dài $AI$ cắt $BC$ tại $L$. Ta có $\widehat{GFI}=\widehat{IAE}=\widehat{BAL}$ và $\widehat{FIG}=\widehat{FIA}+\widehat{AIG}=90^o-\widehat{FAI}+90^o-\widehat{ILD}=180^o-\widehat{FAI}-\widehat{ILD}=\widehat{ABL}$ nên $\triangle FIG \backsim \triangle ABL$, suy ra $\dfrac{FG}{FI}=\dfrac{AL}{AB} \Leftrightarrow AB\cdot FG=AL\cdot FI$. Tương tự $AC\cdot EG=AL\cdot EI$. Do $EI=FI$ nên $AB\cdot FG=AC\cdot EG \Leftrightarrow \dfrac{FG}{AC}=\dfrac{EG}{AB}$. Từ đây ta có:
$$\overrightarrow{AG}=\dfrac{FG}{EF}\overrightarrow{AE}+\dfrac{EG}{EF}\overrightarrow{AF}=\dfrac{FG}{EF}\cdot\dfrac{AE}{AC}\overrightarrow{AC}+\dfrac{EG}{EF}\cdot\dfrac{AF}{AB}\overrightarrow{AB}=k\left(\overrightarrow{AB}+\overrightarrow{AC}\right)$$
Mặt khác do $\overrightarrow{AM}=\dfrac{1}{2}\left(\overrightarrow{AB}+\overrightarrow{AC}\right)$ nên ta được $A$, $G$, $M$ thẳng hàng.
Bổ đề được chứng minh.

Quay trở lại bài toán,

Kéo dài $AB$, $AC$ cắt $EF$ theo thứ tự tại $X$, $Y$. Gọi $K$ là trung điểm của $XY$. Do $PQ \text{ } \bot \text{ } EF$ nên $I$, $P$, $Q$ thẳng hàng. Áp dụng bổ đề cho $\triangle AXY$ được $AQ$ đi qua trung điểm $K$ của $XY$ hay $AQ$ là đường trung tuyến của $\triangle AXY$. Tiếp tục biến đổi góc:
$$\widehat{AYF}=\widehat{EAY}+\widehat{DEF}=\widehat{EAY}+\widehat{EDF}=\widehat{EAY}+\widehat{ADB}=180^o-\widehat{ABC}$$
Do đó tứ giác $BCYX$ nội tiếp hay $BC$ đối song $XY$. Như vậy $AQ$ là đường đối trung của $\triangle ABC$. Suy ra $AQ$ đi qua giao điểm $T$ cố định của tiếp tuyến tại $B$ và $C$ đối với $(O)$. Bài toán được giải quyết hoàn toàn.


Bài viết đã được chỉnh sửa nội dung bởi kimchitwinkle: 16-12-2016 - 08:29


#12
anhquannbk

anhquannbk

    Sĩ quan

  • Thành viên
  • 477 Bài viết

Câu 6: Từ cấu hình bài toán ta chuyển thành bài toán tương đương sau:

Cho $ \bigtriangleup ABC$ nhọn nội tiếp đường tròn $ (O)$, $ H$ là trực tâm tam giác, $ M$ là trung điểm $ BC$, hai tiếp tuyến tại $ B, C$ với $(O)$ cắt nhau tại $ T$, $ R$ là hình chiếu của $ H$ lên tiếp tuyến tại $ A$ với $(O)$

a) Chứng minh rằng $ MR$ và $ AT$ cắt nhau tại một điểm nằm trên đường tròn đường kính $ AH$.

b) $ AT$ cắt (O) tại $ P$. Đường thẳng qua $ P$ vuông góc $ BP$ cắt đường thẳng $ AB$ tại $ M$, đường thẳng qua $ P$ vuông góc $PC$ cắt đường thẳng $ AC$ tại $ N$. CHứng minh $ AM$ là trục đẳng phương của $ (MNP)$ và đường tròn đường kính $ AH$.

a) Gọi $ L$ là giao điểm của $ AT$ và $ (AH)$. Ta sẽ chứng minh $ R, K, M$ thẳng hàng.

Gọi $  AA' $ là đường cao.

$ K$ là hình chiếu của $ H$ lên $AM$, ta có $ K \in (BHC)$

$ AT$ cắt $ BC$ tại $ S$. Khi đó ta có $ L$ là điểm $ Miquel $ của tam giác $ AA'M$, do đó tứ giác $ LKMS$ nội tiếp

Ta có $ KS$ là đối trung trong tam giác $ BKC$ $\implies \angle BKS = \angle MKC$

Gọi $ X, Y$ là giao điểm $ BK, CK$ với $ CA, AB$, ta có $ XY \parallel BC$

và $ \angle XKY =\angle BKC = \angle BHC =180^0-\angle BAC $ $\implies AXKY$ nội tiếp.

$ \angle SLM =\angle SKM =90^0-\angle HKB-\angle BKS =90^0-(90^0-\angle ABC)-\angle CKM = \angle ABC-\angle AXY=\angle ABC- \angle ACB $

Khi đó $ \angle RHM =90^0+\angle RLH + \angle SLM= 90^0+\angle RAH+\angle SLM = 90^0+\angle ABC- \angle ACB +\angle ACB +90^0-\angle ABC= 180^0$

Do đó $ R, H, M$ thẳng hàng, ta có đpcm.

b) Ta có tính chất quen biết $ A, K, P$ thuộc đường tròn $ A-Apolonius$ nên $ (APK)$ tiếp xúc với $ AO$.

$ \implies \angle APK= \angle OAM $

Ta có $ \angle NPK =\angle APK +\angle NAP =\angle OAM +90^0-\angle APC =\angle OAM +90^0-\angle ABC=\angle MAC $

 $\implies  NAKP$ nội tiếp.

Mặt khác $ \angle NPM =\angle BPC =\angle MAN$

$\implies MPAN $ nội tiếp.

Do đó năm điểm $ M, P, N, A, K$ đồng viên.

Từ đây ta có $ AM \equiv AK$ là trục đẳng phương của $ (AH)$ và $ (MNP)$, đpcm. 


Bài viết đã được chỉnh sửa nội dung bởi anhquannbk: 16-12-2016 - 12:24


#13
quanghung86

quanghung86

    Thiếu úy

  • Điều hành viên
  • 632 Bài viết

Một mở rộng cho bài 6 b) trên mô hình mới, có thể thay đường đối trung thành đường thẳng bất kỳ như sau

 

Figure4174.png

 

Cho tam giác $ABC$ nội tiếp đường tròn $(O)$ đường kính $AK$ và $D$ là một điểm nằm trên $(O)$. Trên đường thẳng $CA,AB$ lấy $E,F$ sao cho $DE\perp DC$ và $DF\perp DB$. Đối xứng của $AD$ qua phân giác $\angle BAC$ cắt trung trực $BC$ tại $M$. Trên $KM$ lấy $H$ sao cho $AH\perp BC$. Chứng minh rằng hình chiếu của $H$ trên $AM$ nằm trên đường tròn $(AEFD)$.



#14
kimchitwinkle

kimchitwinkle

    Thiếu úy

  • Điều hành viên THCS
  • 526 Bài viết

Mình xin đưa ra lời giải khác cho Câu 6:
Để ý hai điểm $B$, $C$ không có tác dụng gì ngoài việc tượng trưng $BC$ là tiếp tuyến của $D$ đối với $(I)$ thôi.
Do đó ta viết lại cấu hình bài toán như sau: Cho $\triangle DEF$ nội tiếp $(I)$, trực tâm $H$. Gọi $K$ là hình chiếu của $H$ lên tiếp tuyến tại $D$ của $(I)$. Tiếp tuyến tại $E$ và $F$ của $(I)$ cắt nhau tại $A$.
truongdongvientoan2016day2caua.png
a) Kéo dài $EH$, $FH$ cắt $DF$, $DE$ lần lượt tại $X$, $Y$. Gọi $G$ là giao điểm của $KJ$ với $(T)$. Ta chứng minh $A$, $T$, $D$ thẳng hàng, tức là chứng minh $DT$ là đường đối trung của $\triangle DEF$. Vì $KD \parallel XY \text{ } \bot \text{ } DO$ nên $DKYX$ là hình thang cân. Do $J$ là giao điểm của tiếp tuyến tại $X$ và $Y$ của $(T)$ nên $KG$ là đường đối trung của $\triangle KXY$. Suy ra $\dfrac{GY}{GX}=\dfrac{KY}{KX}=\dfrac{DX}{DY}$. Suy ra $DG$ là đường trung tuyến của $\triangle DXY$. Mặt khác do $XY$ đối song $BC$ nên $DT$ là đường đối trung của $\triangle DEF$. Vậy $A$, $T$, $D$ thẳng hàng. Câu (a) được chứng minh.

b) Gọi $Z$ là giao điểm của $DJ$ và $(T)$. Trước khi chứng minh trọn vẹn bài toán, ta có một bổ đề khá đẹp về mối tương quan giữa trực tâm và đường trung tuyến như sau:

Bổ đề. Cho $\triangle ABC$ nội tiếp $(O)$, trực tâm $H$, $M$ là trung điểm của $BC$. $AM$ cắt $(O)$ tại $P$. Gọi $X$ là điểm đối xứng của $P$ qua $M$. Trên $(O)$ lấy $Q$ sao cho $AQ$ là đường đối trung của $\triangle ABC$. Khi đó $X$ là hình chiếu của $H$ lên $AM$ và $X$ là điểm đối xứng của $Q$ qua $BC$.
truongdongvientoan2016day2bodecaub.png
Chứng minh. Kẻ đường kính $AD$ của $(O)$. Do $BH \parallel CD \text{ } \bot \text{ } AC$ và $CH \parallel BD \text{ } \bot \text{ } AB$ nên tứ giác $BHCD$ là hình bình hành. Suy ra $MH=MD$. Mà $MX=MP$ nên $HXDP$ là hình bình hành. Do đó $HX \parallel PD \text{ } \bot \text{ } AP$ nên $X$ là hình chiếu của $H$ lên $AP$. Ta có $PQ \parallel BC$ nên $BCPQ$ là hình thang cân. Suy ra $MQ=MP=MX$ nên $XQ \text{ } \bot \text{ } PQ \parallel BC$. Từ đó ta kết luận $X$ là điểm đối xứng của $Q$ qua $BC$. Bổ đề được chứng minh.

Quay trở lại bài toán,

Áp dụng bổ đề cho $\triangle DEF$ được $Z$ thuộc $(T)$ và $ZP \parallel AH \text{ } \bot \text{ } BC$. Suy ra:
$$\widehat{DZP}=\widehat{DZH}+\widehat{HZP}=\widehat{DZH}+\widehat{DHZ}=180^o-\widehat{HDZ}=180^o-\widehat{PDI}=180^o-(90^o-\dfrac{1}{2}\widehat{DIP})=180^o-(90^o-\widehat{DFP})=180^o-\widehat{DNP}$$
Do đó $Z$ thuộc $(DNP)$. Tương tự $Z$ thuộc $(DMP)$. Vậy $Z$ thuộc $(MNP)$. Do $Z$, $D$ thuộc $(T)$ và $(MNP)$ nên $DZ \equiv DJ$ là trục đẳng phương của $(T)$ và $(MNP)$. Bài toán được chứng minh hoàn toàn.


Bài viết đã được chỉnh sửa nội dung bởi kimchitwinkle: 17-12-2016 - 12:12


#15
kimchitwinkle

kimchitwinkle

    Thiếu úy

  • Điều hành viên THCS
  • 526 Bài viết

Bài toán của thầy Hùng hoàn toàn có thể giải theo ý tưởng của em, vì thực chất, tính đối trung không còn cần thiết lắm ở câu (b). Thay vào đó là tính song song khá quan trọng $AH \parallel LD$. :)

Câu (b) ở trên là em lấy ý tưởng từ một tính chất trong tài liệu "Điểm cố định" của tác giả Nguyễn Trần Hữu Thịnh tại đây: http://simplegeometr...-quanh-hai.html


Bài viết đã được chỉnh sửa nội dung bởi kimchitwinkle: 17-12-2016 - 20:46


#16
kimchitwinkle

kimchitwinkle

    Thiếu úy

  • Điều hành viên THCS
  • 526 Bài viết

Một cách phát biểu khác tương đương với bài toán của thầy Hùng, ở đây $D$ là điểm bất kì và thay đường trung tuyến bằng đường đẳng giác của $AD$:
truongdongvientoan2016day2morong.png
Cho $\triangle ABC$ nội tiếp $(O)$ và $D$ là một điểm nằm trên $(O)$. Trên đường thẳng $CA$, $AB$ lần lượt lấy $E$, $F$ sao cho $CD \text{ } \bot \text{ } ED$, $BD \text{ } \bot \text{ } FD$. Đường thẳng qua $D$ vuông góc $BC$ cắt đường đẳng giác của $AD$ đối với $\triangle ABC$ tại $L$. Khi đó $L$ nằm trên đường tròn $(ADFE)$.


Bài viết đã được chỉnh sửa nội dung bởi kimchitwinkle: 17-12-2016 - 20:46


#17
manhtuan00

manhtuan00

    Trung sĩ

  • Thành viên
  • 108 Bài viết

lời giải của em cho mở rộng bài hình và bài 7 ạ
Bài 7 : 

Xét với $x = y = z = \frac{1}{3}$ thì ta có $c = \frac{9}{10}$

Ta chứng minh hằng số $c= \frac{9}{10}$ thỏa mãn đề bài

Ta có : $|x^3+y^3+z^3 -1| = |x^3+y^3+z^3 - (x+y+z)^3| = |3(x+y)(y+z)(z+x)|$

Và $|x^5+y^5+z^5-1| = |x^5+y^5+z^5-(x+y+z)^5| = |5(x+y)(y+z)(z+x)(x^2+y^2+z^2+xy+yz+zx)|$

Vậy ta cần chứng minh $|x^2+y^2+z^2+xy+yz+zx| \geq \frac{2}{3}$

Có $x^2+y^2+z^2+xy+yz+zx  = (x+y+z)^2-(xy+yz+zx) \geq \frac{2}{3}$ nên ta có điều cần chứng minh

 

Mở rộng bài 6b :

Gọi $J$ là tâm ngoại tiếp $(ADEF)$
Ta có : $\angle JAD = 90^{\circ} - \angle AED = \angle ACD$ nên $AJ$ là tiếp tuyến của $(O)$
Lấy $T$ đối xứng $A$ qua $J$. Khi đó $\angle ADT = 90^{\circ}$ nên $T,D,K$ thẳng hàng
Ta sẽ chứng minh $TH \perp AM$
Gọi $N$ đối xứng $A$ qua $M, AM$ cắt $(O)$ tại $S$. Khi đó ta có $OM \parallel KM$
nên $KN \perp BC$
$\implies KN \parallel AH \implies \angle KNS = \angle HAN = \angle DAK = \angle ATK$ nên tứ giác $TAKN$
nội tiếp . Từ đây suy ra $KN \perp NT$
Lại có tứ giác $AHNK$ là hình bình hành nên $HN \parallel AK \implies HN \perp AT$
$\implies H$ là trực tâm $\triangle ANT \implies HT \perp AM$
Vậy hình chiếu vuông góc của $H$ lên $AM$ nằm trên đường tròn $(ADEF)$

 

Hình gửi kèm

  • Untitled.png





1 người đang xem chủ đề

0 thành viên, 1 khách, 0 thành viên ẩn danh